Well, it looks symmetric!

Algebra Level 5

The minimum value of ( x 4 + 1 ) ( y 4 + 1 ) ( z 4 + 1 ) x y 2 z \dfrac{(x^4+1)(y^4+1)(z^4+1)}{xy^2z} as x , y , x,y, and z z range over the positive reals is equal to A B C , \dfrac{A\sqrt{B}}{C}, where A A and C C are coprime and B B is squarefree. What is A + B + C ? A+B+C?


The answer is 44.

This section requires Javascript.
You are seeing this because something didn't load right. We suggest you, (a) try refreshing the page, (b) enabling javascript if it is disabled on your browser and, finally, (c) loading the non-javascript version of this page . We're sorry about the hassle.

3 solutions

Daniel Liu
Jun 16, 2014

We see that ( x 4 + 1 ) ( y 4 + 1 ) ( z 4 + 1 ) x y 2 z = ( x 3 + 1 3 x + 1 3 x + 1 3 x ) ( y 2 + 1 y 2 ) ( z 3 + 1 3 z + 1 3 z + 1 3 z ) \dfrac{(x^4+1)(y^4+1)(z^4+1)}{xy^2z}=\left(x^3+\dfrac{1}{3x}+\dfrac{1}{3x}+\dfrac{1}{3x}\right)\left(y^2+\dfrac{1}{y^2}\right)\left(z^3+\dfrac{1}{3z}+\dfrac{1}{3z}+\dfrac{1}{3z}\right)

Using AM-GM on each of these pieces gives ( x 3 + 1 3 x + 1 3 x + 1 3 x ) ( y 2 + 1 y 2 ) ( z 3 + 1 3 z + 1 3 z + 1 3 z ) ( 4 1 27 4 ) ( 2 ) ( 4 1 27 4 ) = 32 3 9 \left(x^3+\dfrac{1}{3x}+\dfrac{1}{3x}+\dfrac{1}{3x}\right)\left(y^2+\dfrac{1}{y^2}\right)\left(z^3+\dfrac{1}{3z}+\dfrac{1}{3z}+\dfrac{1}{3z}\right)\ge \left(4\sqrt[4]{\dfrac{1}{27}}\right)(2)\left(4\sqrt[4]{\dfrac{1}{27}}\right)=\dfrac{32\sqrt{3}}{9}

so our answer is 32 + 3 + 9 = 44 32+3+9=\boxed{44} .

Equality case is when ( x , y , z ) = ( 1 3 4 , 1 , 1 3 4 ) (x,y,z)=\left(\dfrac{1}{\sqrt[4]{3}},1,\dfrac{1}{\sqrt[4]{3}}\right)

Thanks to @Nathan Ramesh for giving me the crucial inspiration for this problem (although I think he is still stuck on it at this moment, lol ¨ \ddot\smile ).

Before, for some reason, I was expanding and trying to mega AM-GM bash when I knew all along that it wouldn't work because the degrees weren't right.

Daniel Liu - 6 years, 12 months ago

It's slightly easier to see and approach it as in @Jubayer Nirjhor 's solution.

Variable separable inequalities generally should be compartmentalized.

Calvin Lin Staff - 6 years, 11 months ago
Jubayer Nirjhor
Jun 17, 2014

Note that by AM-GM inequality, y 4 + 1 2 y 2 y^4+1\geq 2y^2 so we can cancel the y 2 y^2 . This trick doesn't work for others. So we split up the 1 1 so that we have 4 4 terms to cancel the 4 4 th power. x 4 + 1 = x 4 + 1 3 + 1 3 + 1 3 4 x 4 3 3 4 = 4 x 3 3 4 x^4 + 1 = x^4 + \dfrac{1}{3} +\dfrac{1}{3}+\dfrac{1}{3} \geq 4\sqrt[4]{\dfrac{x^4}{3^3}} = \dfrac{4x}{\sqrt[4]{3^3}} z 4 + 1 = z 4 + 1 3 + 1 3 + 1 3 4 z 4 3 3 4 = 4 z 3 3 4 z^4+1=z^4+\dfrac 1 3+\dfrac 1 3+\dfrac 1 3\geq 4\sqrt[4]{\dfrac{z^4}{3^3}}=\dfrac{4z}{\sqrt[4]{3^3}}

Therefore: ( x 4 + 1 ) ( y 4 + 1 ) ( z 4 + 1 ) x y 2 z 4 x 3 3 4 2 y 2 4 z 3 3 4 1 x y 2 z = 32 3 9 \dfrac{(x^4+1)(y^4+1)(z^4+1)}{xy^2z}\geq \dfrac{4x}{\sqrt[4]{3^3}}\cdot 2y^2\cdot \dfrac{4z}{\sqrt[4]{3^3}}\cdot \dfrac{1}{xy^2 z}=\dfrac{32\sqrt{3}}{9}

Thus the answer is 44 \fbox{44} .

Note: the equality occurs when y 4 = 1 y = 1 y^4=1\implies y=1 , x 4 = z 4 = 1 / 3 x^4=z^4=1/3 x = z = 1 / 3 4 \implies x=z=1/\sqrt[4]{3} .

Himanshu Arora
Jun 17, 2014

We see that x ( ( x 4 + 1 ) ( y 4 + 1 ) ( z 4 + 1 ) x y 2 z ) = 0 \frac { \partial }{ \partial x } \left( \frac { ({ x }^{ 4 }+1)({ y }^{ 4 }+1)({ z }^{ 4 }+1) }{ x{ y }^{ 2 }z } \right) = 0 for x = 3 1 / 4 x = 3^{1/4} and y ( ( x 4 + 1 ) ( y 4 + 1 ) ( z 4 + 1 ) x y 2 z ) = 0 \frac { \partial }{ \partial y } \left( \frac { ({ x }^{ 4 }+1)({ y }^{ 4 }+1)({ z }^{ 4 }+1) }{ x{ y }^{ 2 }z } \right) = 0 for y = 1 y=1 . The equation is symmetric in x x and z z , and the functions are increasing in x , y x, y and z z . Hence, the minimum value is 32 3 9 \frac{32\sqrt3}{9} . Which gives us the answer 44 \boxed{44}

0 pending reports

×

Problem Loading...

Note Loading...

Set Loading...